A 72.5 g sample of a metal, initial temperature at 100.0 °C, is mixed with 200.0 mL of water, initially at 24.0 °C, in a calorimeter. The water/metal mixture reaches an equilibrium temperature of 26.8 °C. Calculate the specific heat of this metal.

Answers

Answer 1

A 72.5 g sample of a metal, initial temperature at 100.0 °C, is mixed with 200.0 mL of water, initially at 24.0 °C, in a calorimeter. The water/metal mixture reaches an equilibrium temperature of 26.8 °C. The specific heat of this metal is 11.53 J/g °C.

To calculate the specific heat of the metal, we can use the principle of energy conservation. The heat lost by the metal is equal to the heat gained by the water in the calorimeter.

The heat lost by the metal can be calculated using the formula:

[tex]Q_l_o_s_s[/tex] = m*c*ΔT

Where: [tex]Q_l_o_s_s[/tex] is the heat lost by the metal

m is the mass of the metal (72.5 g)

c is the specific heat of the metal (unknown)

ΔT is the change in temperature of the metal (26.8 °C - 100.0 °C)

The heat gained by the water can be calculated using the formula:

[tex]Q_g_a_i_n=m_w_a_t_e_r *c_w_a_t_e_r[/tex] * ΔT

Where: [tex]Q_g_a_i_n[/tex] is the heat gained by the water

[tex]m_w_a_t_e_r[/tex] is the mass of the water

[tex]c_w_a_t_e_r[/tex] is the specific heat of water (4.18 J/g °C)

ΔT is the change in temperature of the water (26.8 °C - 24.0 °C)

Since the heat lost by the metal is equal to the heat gained by the water,  then:

m * c * ΔT = [tex]m_w_a_t_e_r *c_w_a_t_e_r[/tex]* ΔT

We can cancel out the ΔT terms:

m * c = [tex]m_w_a_t_e_r*c_w_a_t_e_r[/tex]

72.5 g * c = 200.0 g * 4.18 J/g °C

c = (200.0 g * 4.18 J/g° C) / 72.5 g

c ≈ 11.53 J/g °C

Therefore, the specific heat of the metal is approximately 11.56 J/g °C.

To know more about  specific heat here

https://brainly.com/question/31608647

#SPJ4


Related Questions

The base of a box is .45 m by .65 m. It weighs 8694 N. What is the pressure exerted on the floor by the box?

Answers

Answer:

[tex]Pressure = 29723.1\ N/m^2[/tex]

Explanation:

Given

[tex]Force = 8694N[/tex]

[tex]Length = 0.45m[/tex]

[tex]Width = 0.65m[/tex]

Required

The force exerted on the floor by the box

First, calculate the area covered by the box (i.e. the base area)

[tex]Base\ Area = Length * Width[/tex]

[tex]Base\ Area = 0.45m * 0.65m[/tex]

[tex]Base\ Area = 0.2925m^2[/tex]

Pressure is calculated as:

[tex]Pressure = \frac{Force}{Area}[/tex]

[tex]Pressure = \frac{8694N}{0.2925m^2}[/tex]

[tex]Pressure = 29723.0769231\ N/m^2[/tex]

[tex]Pressure = 29723.1\ N/m^2[/tex] --- approximated

A quasar is observed with a redshift of z = 3.7. What will be the wavelength of the observed Lyman-alpha line from this quasar? wavelength: ___ nm
This is in the visible part of the clectromagnetic spectrum.

Answers

The wavelength of the observed Lyman-alpha line from this quasar with a redshift of z = 3.7 will be approximately 444 nm.

The formula to calculate the observed wavelength (λ_obs) of an object with redshift (z) is given by:

λ_obs = λ_rest * (1 + z)

In this case, we are interested in the Lyman-alpha line, which has a rest wavelength (λ_rest) of 121.6 nm.

Substituting the values into the formula, we have:

λ_obs = 121.6 nm * (1 + 3.7)

λ_obs = 121.6 nm * 4.7

λ_obs ≈ 570.32 nm

However, it's important to note that the Lyman-alpha line falls in the ultraviolet (UV) region of the electromagnetic spectrum, not in the visible part.

Since the question specifically mentions that it is in the visible part, it suggests a typographical error or some other contextual discrepancy.

The wavelength of the observed Lyman-alpha line from this quasar, assuming a redshift of z = 3.7, is approximately 570.32 nm.

However, the Lyman-alpha line is in the ultraviolet (UV) region, not the visible part of the electromagnetic spectrum.

To know more about quasar visit:

https://brainly.com/question/20308478

#SPJ11

A tank contains 2 m3 of air at -93°C and a gage pressure Ro6 of 1.4 MPa. Determine the mass of air, in kg. The local atmospheric pressure is 1 atm

Answers

To determine the mass of air in the tank, we need to convert the given parameters and apply the ideal gas law. The mass of air in the tank is approximately 5.04 kg.

To determine the mass of air in the tank, we need to consider the ideal gas law and convert the given parameters to appropriate units.

Given:

Volume of air (V) = 2 m³

Temperature (T) = -93°C

Gauge pressure (P) = 1.4 MPa

Local atmospheric pressure (P_atm) = 1 atm

First, let's convert the temperature from Celsius to Kelvin:

T = -93°C + 273.15 = 180.15 K

Next, we need to convert the gauge pressure to absolute pressure by adding the atmospheric pressure:

P_abs = P + P_atm = 1.4 MPa + 1 atm = 2.4 MPa

Now, we can use the ideal gas law equation to calculate the mass of air (m):

PV = nRT

Where:

P = absolute pressure

V = volume

n = number of moles of air

R = ideal gas constant

T = temperature

Rearranging the equation to solve for mass (m):

m = (n * M) / N_A

Where:

M = molar mass of air

N_A = Avogadro's number

To find the number of moles (n), we can use the equation:

n = PV / RT

Given that the molar mass of air is approximately 28.97 g/mol, and the ideal gas constant R is 8.314 J/(mol·K), we can calculate the mass of air.

Calculations:

n = (P_abs * V) / (R * T)

m = (n * M) / N_A

Substituting the values:

n = (2.4 MPa * 2 m³) / (8.314 J/(mol·K) * 180.15 K)

m = (n * 28.97 g/mol) / 6.022 x 10^23 mol⁻¹

Calculating the mass of air (m):

m ≈ 5.04 kg

Therefore, the mass of air in the tank is approximately 5.04 kg.

To know more about parameters, click here https://brainly.com/question/29728062

#SPJ11

The only players permitted to wear mitts over gloves are _________.
1 the pitcher and first base player
2 the catcher and outfielder
3 the catcher and first base player
4 the catcher and pitcher

Answers

Answer:

the catcher and outfielder

Explanation:

Answer: The catcher and the player at first base are the only players permitted to wear mitts rather than gloves. So, the answer would be 3.

Explanation: I had this on a quiz and got it right.

Hope this helps!

A satellite of mass 180 kg is placed into Earth orbit at a height of 100 km above the surface. (a) Assuming a circular orbit, how long does the satellite take to complete one orbit?
1.47
h
(b) What is the satellite's speed?
7430
m/s
(c) Starting from the satellite on the Earth's surface, what is the minimum energy input necessary to place this satellite in orbit? Ignore air resistance but include the effect of the planet's daily rotation.
7.33e+09
J

Answers

(a) The satellite takes approximately 1.47 hours to complete one orbit.

(b) The satellite's speed is approximately 7430 m/s.

(c) The minimum energy input necessary to place the satellite in orbit, starting from the Earth's surface, is approximately 7.33 x 10^9 J.

Determine how the satellite take to complete one orbit?

(a) To determine the time taken to complete one orbit, we can use Kepler's third law, which states that the square of the orbital period (T) is proportional to the cube of the semi-major axis (a) of the orbit.

Since the satellite is in a circular orbit, the semi-major axis is equal to the radius of the orbit, which is the sum of the Earth's radius (R) and the height above the surface (h). Using the given values, we can calculate T as follows:

T² = (4π² / GM) * a³

T² = (4π² / GM) * (R + h)³

T = √[(4π² / GM) * (R + h)³]

Substituting the known values, we find T ≈ 1.47 hours.

Determine how to find the satellite speed?

(b) The speed of the satellite can be determined using the formula for orbital speed:

v = √(GM / r)

where G is the gravitational constant, M is the mass of the Earth, and r is the distance from the center of the Earth to the satellite's orbit (R + h). Plugging in the values, we find v ≈ 7430 m/s.

Determine what is the minimum energy input?

(c) The minimum energy input required to place the satellite in orbit can be calculated as the sum of the gravitational potential energy and the kinetic energy of the satellite. The gravitational potential energy is given by:

PE = -GMm / r

where m is the mass of the satellite and r is the distance from the center of the Earth to the satellite's orbit (R + h). The kinetic energy is given by:

KE = 0.5mv²

Plugging in the values, we find the minimum energy input is approximately 7.33 x 10^9 J.

To know more about potential energy, refer here:

https://brainly.com/question/24284560#

#SPJ4

An electromagnetic wave with frequency 65.0Hz travels in an insulating magnetic material that has dielectric constant 3.64 and relative permeability 5.18 at this frequency. The electric field has amplitude 7.20×10−3V/m. What is the wavelength of the wave?

Answers

Answer:

  The wavelength of the wave is [tex]1.06\times10^6 m[/tex]  

Explanation:

Lets calculate

We know an electromagnetic wave is propagating through an insulating magnetic material of dielectric constant K and relative permeability [tex]K_m[/tex] ,then the speed of the wave in this dielectric medium is [tex]\nu[/tex] is less than the speed of the light c and is given by a relation

               [tex]\nu=\frac{c}{\sqrt{KK_m} }[/tex]  --------- 1

In case the electromagnetic  wave propagating through the insulating magnetic material , the amplitudes of electric and  magnetic fields are related as -

             [tex]E_m_a_x= \nu B_m_a_x[/tex]

The magnitude of the 'time averaged value' of the pointing vector is called the intensity of the wave and is given by a relation

                       [tex]I = S_a_v[/tex]

                        [tex]\frac{E_m_a_xB_m_a_x}{2K_m\mu0}[/tex]----------- 3

now , we will find the speed of the propagation of an electromagnetic wave by using equation 1

[tex]\nu=\frac{c}{\sqrt{KK_m} }[/tex]

Putting the values ,

   =[tex]\nu= \frac{3.00\times10^8}{\sqrt{(3.64)(5.18)} }[/tex]

 =[tex]0.6908\times10^8m/s[/tex]

 = [tex]6.91\times10^7m/s[/tex]

Now , using this above solution , we will find the wavelength of the wave -

     [tex]\lambda=\frac{\nu}{f}[/tex]

    Putting the values from above equations -

  [tex]\frac{6.91\times10^7m/s}{65.0Hz}[/tex]

        [tex]\lambda= 1.06\times10^6 m[/tex]

Hence , the answer is [tex]\lambda= 1.06\times10^6 m[/tex]

A 200-m radio telescope is used to investigate sources emitting a 21-cm wavelength. The
minimum angular separation resolvable for this system is
Select one:
a. 0.073°
b. 0.154°
c. 0.0013°
d. 0.0026°
e. 0.03°

Answers

The minimum angular separation resolvable for a 200-m radio telescope investigating sources emitting a 21-cm wavelength is 0.073°.

The angular resolution of a telescope is determined by the ratio of the wavelength of the radiation being observed to the diameter of the telescope. In this case, the telescope has a diameter of 200 meters, and the wavelength being observed is 21 cm (or 0.21 m).

The formula for calculating the angular resolution is given by θ = λ/D, where θ is the angular resolution, λ is the wavelength, and D is the diameter of the telescope. Substituting the given values into the formula, we get θ = 0.21 m / 200 m = 0.00105 radians.

To convert this to degrees, we multiply by (180/π), which gives us approximately 0.073°. Therefore, the minimum angular separation resolvable for this system is 0.073°.

Learn more about angular resolution here:

https://brainly.com/question/31858807

#SPJ11

An object with mass 2.7 kg is executing simple harmonic motion, attached to a spring with spring constant k = 310 N/m. When the object is 0.020 m from its equilibrium position, it is moving with a speed of 0.55 m/s. (a) Calculate the amplitude of the motion. (b) Calculate the maximum speed attained by the object.
a.The amplitude of the motion is \(0.02\;{\rm{m}}\).
bThe maximum speed attained by the object is \(0.352\;{{\rm{m}} \mathord{\left/{\vphantom {{\rm{m}} {\rm{s}}}} \right.} {\rm{s}}}\).

Answers

The amplitude of the motion is 0.02 m. The maximum speed attained by the object is 0.352 m/s.

a.

In simple harmonic motion (SHM), the displacement of an object from its equilibrium position can be described by the equation:

x(t) = A * cos(ωt + φ)

where:

x(t) is the displacement at time t,

A is the amplitude of the motion,

ω is the angular frequency,

t is the time, and

φ is the phase constant.

The speed of the object is given by:

v(t) = A * ω * sin(ωt + φ)

Mass of the object, m = 2.7 kg

Spring constant, k = 310 N/m

Displacement from equilibrium position, x = 0.02 m

Speed of the object, v = 0.55 m/s

We can relate the angular frequency (ω) to the mass and spring constant using the equation:

ω = sqrt(k / m)

Let's calculate ω first:

ω = sqrt(310 N/m / 2.7 kg) ≈ 8.064 rad/s

To find the amplitude (A), we can use the given displacement:

0.02 m = A * cos(0 + φ)

cos(0) = 1

Therefore, we have:

A = 0.02 m

The amplitude of the motion is 0.02 m.

b.

The maximum speed in simple harmonic motion occurs when the displacement is zero (i.e., at the equilibrium position). At this point, the speed is at its maximum value.

Amplitude of the motion, A = 0.02 m

We can find the maximum speed (v_max) using the equation:

v_max = A * ω

Substituting the values:

v_max = 0.02 m * 8.064 rad/s ≈ 0.352 m/s

Conclusion:

The maximum speed attained by the object is 0.352 m/s.

To know more about amplitude visit:

https://brainly.com/question/3613222

#SPJ11

A wave created by a certain source travels from medium 1 into another medium 2. It is noticed that its velocity is faster in medium 2 than in medium 1. Three students are discussing what happens to the properties of the wave as it moves into medium 2. Student 1: The frequency of this wave increases as this wave moves into medium 2 in order to keep the equation of the velocity of a wave valid. Student 2: No, the frequency of the wave will remain the same as it is only dependent on the source, it will be the wavelength that will increase in order to keep the equation of the velocity of a wave valid. Student 3: No, you are both wrong. Both parameters will adjust in order to keep the equation of the velocity of a wave valid. Which one of these students do you agree with? Justify your response with words and or equations.

Answers

I agree with Student 2: The frequency of the wave will remain the same as it is only dependent on the source, while the wavelength will increase as the wave moves into medium 2.

The equation that relates the velocity (v), frequency (f), and wavelength (λ) of a wave is:

v = f * λ

According to this equation, if the velocity increases in medium 2 compared to medium 1, and the frequency remains constant (as stated by Student 2), then the only way to maintain the equation is for the wavelength to increase in medium 2.

This behavior can be explained by the fact that different media have different properties, such as density and elasticity, which affect the propagation of the wave. When a wave travels from one medium to another, the speed of the wave can change. However, the frequency of the wave is determined by the source and remains constant. Therefore, in order to maintain the equation v = f * λ, the wavelength must adjust to compensate for the change in velocity.

In summary, Student 2 is correct in stating that the frequency of the wave will remain the same, while the wavelength will increase as the wave moves into medium 2 to keep the equation of the wave velocity valid.

To know more about frequency here

https://brainly.com/question/14316711

#SPJ4

A laser produces light of wavelength 615 nm in an ultrashort pulse.
What is the minimum duration of the pulse if the minimum uncertainty in the energy of the photons is 1.0%?

Answers

The minimum duration of the pulse is approximately 3.27 femtoseconds. This calculation is based on the uncertainty principle and the given uncertainty in energy, wavelength, and Planck's constant.

According to the uncertainty principle in quantum mechanics, there is a fundamental limit to the precision with which certain pairs of physical properties, such as energy and time, can be simultaneously known. In the case of light, the uncertainty principle relates the uncertainty in energy (∆E) to the uncertainty in time (∆t) through the equation:

∆E ∆t ≥ h/2π

where ∆E is the uncertainty in energy, ∆t is the uncertainty in time, and h is Planck's constant (approximately 6.626 × 10^(-34) J·s).

We are given the uncertainty in energy as 1.0% of the total energy of the photons. This can be expressed as:

∆E = 0.01 × E

where E is the total energy of the photons.

The energy of a photon can be calculated using the equation:

E = hc/λ

where h is Planck's constant, c is the speed of light in a vacuum (approximately 3.0 × 10^8 m/s), and λ is the wavelength of the light.

Substituting the given wavelength into the equation:

E = (6.626 × 10^(-34) J·s × 3.0 × 10^8 m/s) / (615 × 10^(-9) m)

E ≈ 3.22 × 10^(-19) J

Substituting the value of ∆E into the uncertainty principle equation:

0.01 × E ∆t ≥ h/2π

0.01 × (3.22 × 10^(-19) J) ∆t ≥ (6.626 × 10^(-34) J·s) / (2π)

0.01 × (3.22 × 10^(-19) J) ∆t ≥ 1.05 × 10^(-34) J·s

∆t ≥ (1.05 × 10^(-34) J·s) / (0.01 × 3.22 × 10^(-19) J)

∆t ≥ 3.27 × 10^(-15) s

To convert the time to femtoseconds (fs), we multiply by 10^15:

∆t ≈ 3.27 fs

Therefore, the minimum duration of the pulse is approximately 3.27 femtoseconds.

The minimum duration of the pulse is approximately 3.27 femtoseconds. This calculation is based on the uncertainty principle and the given uncertainty in energy, wavelength, and Planck's constant.

To know more about Planck's constant visit:

https://brainly.com/question/30991884

#SPJ11

A proton traveling due north enters a region that contains both a magnetic field and an electric field. The electric field lines point due west. It is observed that the proton continues to travel in a straight line due north. In which direction must the magnetic field lines point?
A. East
B. West
C. Into Page
D. Out of Page
E. South

Answers

The magnetic field lines pοint tοwards the East (A).

What are magnetic field lines?  

Magnetic field lines are a visual representatiοn used tο depict the directiοn and strength οf the magnetic field arοund a magnet οr a current-carrying cοnductοr. They indicate the path that a hypοthetical magnetic nοrth pοle wοuld take if placed in the vicinity οf the magnetic field.

The prοtοn is a pοsitively charged particle and is traveling due nοrth. Since the electric field lines pοint due west, the electric fοrce οn the prοtοn is directed tοwards the west. In οrder fοr the prοtοn tο cοntinue traveling in a straight line due nοrth, the magnetic fοrce οn the prοtοn must be directed tοwards the east. This can be achieved if the magnetic field lines pοint tοwards the east.

Therefοre, the magnetic field lines pοint tοwards the East directiοn which is οptiοn A.

To learn more about magnetic field lines,

https://brainly.com/question/17011493

#SPJ4

true/false. the sun radiates at an effective temperature of 5780 k and has a radius of about 696000 km

Answers

True. The Sun radiates at an effective temperature of 5780 K and has a radius of about 696,000 km.

The Sun's effective temperature refers to the temperature at which a black body (an idealized object that absorbs all incident radiation) would emit the same amount of radiation as the Sun. This value is approximately 5780 K, representing the temperature of the Sun's photosphere, the visible surface. Regarding the Sun's radius, it has an estimated average radius of about 696,000 km. This measurement defines the distance from the center of the Sun to its outer edge. The Sun is classified as a G-type main-sequence star, and its size is relatively large compared to other stars, making it an important reference point for understanding stellar characteristics.

To learn more about effective temperature, Click here:

https://brainly.com/question/30799008

#SPJ11

an electron is in the ground state of an infinite square well. the energy of the ground state is e1 = 0.86 ev. use hc=1240 nm ev.
(a) What wavelength of electromagnetic radiation would be needed to excite the electron to the n = 3 state?
nm
(b) What is the width of the square well?
nm

Answers

a) The wavelength of electromagnetic radiation needed to excite the electron to the n = 3 state is approximately 1808.26 nm.

b) The width of the square well is approximately 904.13 nm.

To find the wavelength of electromagnetic radiation needed to excite the electron to the n = 3 state, we can use the formula:

λ = hc / ΔE

where λ is the wavelength, h is Planck's constant (6.626 × 10⁻³⁴ J·s), c is the speed of light (2.998 × 10⁸ m/s), and ΔE is the energy difference between the two states.

First, let's convert the energy difference ΔE from electron volts (eV) to joules (J):

ΔE = (3² - 1²) x e₁

ΔE = (9 - 1) x 0.86 eV

ΔE = 8 x 0.86 eV

ΔE = 6.88 eV

Next, let's convert the energy difference ΔE from eV to joules (J):

1 eV = 1.602 × 10⁻¹⁹ J

ΔE = 6.88 x 1.602 × 10⁻¹⁹ J

ΔE ≈ 1.101376 × 10⁻¹⁸ J

Now we can calculate the wavelength λ:

λ = (hc) / ΔE

λ = (6.626 × 10⁻³⁴ J·s x 2.998 × 10⁸ m/s) / (1.101376 × 10⁻¹⁸ J)

λ ≈ 1.80826 × 10⁻⁶ m

Finally, let's convert the wavelength from meters (m) to nanometers (nm):

1 m = 1 × 10⁹ nm

λ ≈ 1.80826 × 10⁻⁶ m x 1 × 10⁹ nm/m

λ ≈ 1808.26 nm

Therefore, the wavelength of electromagnetic radiation needed to excite the electron to the n = 3 state is approximately 1808.26 nm.

To find the width of the square well, we can use the formula:

L = λ / 2

where L is the width of the square well.

Using the value we calculated for λ in part (a):

L = 1808.26/2 nm

L = 904.13 nm

Therefore, the width of the square well is approximately 904.13 nm.

To know more about electromagnetic radiation follow

https://brainly.com/question/27431416

#SPJ4

Block 1 is attached to a spring and oscillates on a horizontal frictionless surface. When block 1 is at a point of maximum displacement, block 2 is placed on top of it from directly above without interrupting the oscillation, and the two blocks stick together. How do the maximum kinetic energy and period of oscillation with both blocks compare to those of block 1 alone?
a. Maximum Kinetic energy - smaller, Period - smaller
b. Maximum Kinetic energy - smaller, Period - greater
c. Maximum Kinetic energy - The same, Period - smaller
d. Maximum Kinetic energy - The same, Period - greater

Answers

The correct answer is: a. Maximum Kinetic energy - smaller, Period - smaller. When block 2 is placed on top of block 1 without interrupting the oscillation, the system's mass increases.

As a result, the effective mass of the combined blocks increases, which leads to a decrease in the maximum kinetic energy of the system. This is because the kinetic energy is directly proportional to the mass of the system. Additionally, the period of oscillation is determined by the mass and the spring constant of the system. With an increase in the combined mass of the blocks, the period of oscillation becomes smaller. This is because the effective mass affects how quickly the system can oscillate back and forth, and a larger mass requires more time to complete each oscillation. Therefore, when block 1 and block 2 stick together, the maximum kinetic energy decreases compared to block 1 alone, and the period of oscillation also decreases.

To learn more about Kinetic energy, Click here:

https://brainly.com/question/16012885

#SPJ11

Which of the following is another name for a convex lans?
O Diverging liens
3. Converging lens
O c Shrinking lans
OD Security lans

Answers

I am going with D hope that helps

Answer:

3. converging lens

Explanation:

When the rays of light coming parallel to principle axis after refraction through the lens passes through a point called focus, since it converge all the ray at one point, that is why it is said to be converging lens.

Explain why, in terms of forces, there is a risk of head injury when diving from this height. Suggest why the high divers would choose to enter the water feet first.

Answers

Answer:

Due to lower risk of injury or damage.

Explanation:

The high divers would choose to enter the water from the feet first because there is low risk of injury. The brain is the most important part of the body which very sensitive to any small injury. Small injury to brain leads to big problems in life. High divers can reach speeds of nearly 60 mph and enters about 28m into the water in about three seconds which can damage the head region if comes in contact with the ground so this is the reason the high divers avoid of entering in the water through their heads and choose entering through their feet.

1. Find the density of the N nucleus. 2. The binding energy per nucleon E, of the lithium isotope Li is 5.6 MeV/nucleon. Find its atomic mass of this isotope. 3. Find the energy needed to remove a proton from the nucleus of the potassium isotopek

Answers

The density of the N nucleus can be calculated by dividing its mass by its volume, the binding energy per nucleon E of the lithium isotope Li is [tex]5.6 MeV/nucleon[/tex], the energy needed to remove a proton from the nucleus of the potassium isotope K is [tex]289.77 MeV[/tex]

The mass number of N is 14 and its atomic number is 7. The number of neutrons in the N nucleus is given by [tex]14 - 7 = 7[/tex] neutrons.

The mass of one neutron is about 1.008665 atomic mass units (amu) or [tex]1.67493 \times 10^{-27} kg[/tex].

The mass of the N nucleus = [tex]7(1.008665) + 7.016004 = 14.04273 $ amu[/tex]. Thus, the density of the N nucleus can be calculated by dividing its mass by its volume.

The binding energy per nucleon E of the lithium isotope Li is 5.6 MeV/nucleon. To find its atomic mass of this isotope, the mass defect of the nucleus is calculated using the formula:

Mass defect = (Zmp + Nmn) - M

where

Z = number of protons, N = number of neutrons, mp = mass of a proton, mn = mass of a neutron, M = mass of the nucleus.

The mass of a proton is approximately [tex]1.00728 amu[/tex], while the mass of a neutron is approximately [tex]1.00866 amu[/tex].

Mass defect = [tex](3 \times 1.00728 + 4 \times 1.00866) - 7.01600[/tex]Mass defect = [tex]0.126 $ amu[/tex]Atomic mass of [tex]Li-7 = 7.01600 - 0.126[/tex]Atomic mass of [tex]Li-7 = 6.89 amu[/tex]

The energy needed to remove a proton from the nucleus of the potassium isotope K can be calculated using the formula:

Binding energy = [tex]E \times A[/tex]

where E is the binding energy per nucleon, A is the mass number. Binding energy of K isotope = 7.43 MeV/nucleon (given)

Mass number of K isotope = [tex]39[/tex]Binding energy = [tex]7.43 \times 39[/tex]Binding energy = [tex]289.77 MeV[/tex]

Thus, the energy needed to remove a proton from the nucleus of the potassium isotope K is [tex]289.77 MeV[/tex].

Learn more about The density: brainly.com/question/1354972

#SPJ11

uranus and neptune are like the other jovian planets because they:

Answers

Uranus and Neptune are like the other Jovian planets because they have a lot of gaseous composition, they lack a solid surface, and they are situated far from the sun.

The Jovian planets are four planets in the outer solar system that are gas giants, also known as the gas planets. They are Jupiter, Saturn, Uranus, and Neptune. In terms of the composition of Uranus and Neptune, they contain methane, ammonia, water, and other elements that form gas. Like the other Jovian planets, they lack a solid surface, and they are situated far from the sun.

The rings of Uranus and Neptune are fainter and less complex than the rings of Jupiter and Saturn, but they share certain features. Both Uranus and Neptune are thought to have a small rocky core surrounded by a mix of rock and ice, then a thick layer of metallic hydrogen, and an atmosphere mainly of molecular hydrogen and helium. So therefore because gaseous composition, lack a solid surface, and situated far from the sun, Uranus and Neptune are like the other Jovian planets.

To know more about Jovian planets visit:

https://brainly.com/question/31384996

#SPJ11

Two long, parallel wires carry currents of I1 = 27.0 A and I2 = 13.5 A in opposite directions (see figure below). Which of the following statements must be true? More than one statement may be correct.
In region I, the magnetic field is into the page and is never zero.
In region II, the field is into the page and can be zero.
In region III, it is possible for the field to be zero.
In region I, the magnetic field is out of the page and is never zero.
There are no points where the field is zero.

Answers

The following statements are true:

- In region I, the magnetic field is into the page and is never zero.

- In region II, the field is into the page and can be zero.

- In region III, it is possible for the field to be zero.

The magnetic field produced by a current-carrying wire follows the right-hand rule. When two parallel wires carrying currents in opposite directions are considered, the magnetic field in the regions around the wires can be determined.

In region I, the magnetic field is between the two wires. According to the right-hand rule, the magnetic field produced by the current in wire I1 is into the page, while the field produced by the current in wire I2 is out of the page. These fields add up to create a net magnetic field into the page. Since the currents are non-zero, the magnetic field in region I is never zero.

In region II, the magnetic field is outside the wires. The fields produced by the currents in wires I1 and I2 are still into the page and out of the page, respectively. However, at certain points between the wires, the magnitudes of these fields can cancel each other out, resulting in a net magnetic field of zero. Therefore, in region II, the field can be zero.

In region III, which is outside both wires, the magnetic field produced by each wire individually decreases with distance. At a certain distance from the wires, the magnetic fields can cancel each other out, resulting in a net magnetic field of zero. Therefore, in region III, it is possible for the magnetic field to be zero.

In summary, in the given scenario of two long, parallel wires carrying currents in opposite directions, the magnetic field is into the page in region I and can be zero in regions II and III. This understanding is based on the right-hand rule and the superposition principle for magnetic fields.

To know more about magnetic field, visit

https://brainly.com/question/14411049

#SPJ11

The moon has no atmosphere. Predict what would happen if an astronaut oro
hammer and a feather at the same time from the same height.

Answers

Answer:

They would all most likely have the same weight

Explanation:

Think about it the smaller the person or object the smaller the weight. But theyre all the same height now. And its not like you don't have any weight in the moon. So in conclusion they would all be the same weight

An amusement park builds a ride in which the victim is made to spin about a pole with a rocket strapped on his seat. The rider, box and rocket have an initial total mass of 170 kg. Neglect the mass of the rod of length 6 m .What is the moment of inertia of rider, box and rocket about the pole? The acceleration of gravity is 9.8 m/s^2 . Treat the rider, box and rocket as a point mass. Answer in units of kg · m^2
B) The rocket develops a thrust of 98 N perpendicular to the path of the rider. What is the initial angular acceleration of the rider? Answer in units of rad/s^2
C)After what time t is the rider’s velocity equal to 5 m/s? Neglect the change in mass of the rocket. Answer in units of s.
D)Gas exits the rocket at vt = 390 m/s. What mass per second must leave to develop the thrust F given above? Answer in units of kg/s .

Answers

The moment of inertia of the rider, box, and rocket about the pole is 170 kg * (6 m)^2 = 6120 kg · m^2.  The initial angular acceleration of the rider is 0.098 rad/s^2.

A) The moment of inertia of the rider, box, and rocket about the pole can be calculated using the formula for the moment of inertia of a point mass rotating around an axis. The moment of inertia (I) is given by the product of the mass (m) and the square of the distance (r) from the axis of rotation:

I = m * r^2

Since the mass of the rider, box, and rocket is given as 170 kg and the rod is neglected, the moment of inertia will be the same for all components. Therefore, the moment of inertia of the rider, box, and rocket about the pole is 170 kg * (6 m)^2 = 6120 kg · m^2.

B) The initial angular acceleration of the rider can be determined using Newton's second law of rotational motion, which states that the torque (τ) is equal to the moment of inertia (I) multiplied by the angular acceleration (α). The torque can be calculated as the product of the force (F) applied perpendicular to the path of the rider and the distance (r) from the axis of rotation:

τ = F * r

Rearranging the formula and substituting the given values:

α = τ / I = (98 N * 6 m) / 6120 kg · m^2 = 0.098 rad/s^2

Therefore, the initial angular acceleration of the rider is 0.098 rad/s^2.

C) The relationship between angular velocity (ω) and time (t) is given by the equation ω = α * t. Rearranging the formula to solve for time:

t = ω / α

The rider's velocity can be converted to angular velocity using the formula v = r * ω. Rearranging the formula to solve for angular velocity:

ω = v / r = 5 m/s / 6 m = 0.8333 rad/s

Substituting the values into the equation, we get:

t = 0.8333 rad/s / 0.098 rad/s^2 = 8.5 s

Therefore, the time required for the rider's velocity to reach 5 m/s is 8.5 seconds.

D) To find the mass per second that must leave the rocket to develop the given thrust (F), we can use Newton's second law of motion, which states that force (F) is equal to the rate of change of momentum (dp/dt). In this case, the force is equal to the thrust of the rocket, and the change in momentum is the mass per second leaving the rocket (dm/dt) multiplied by the exit velocity (v):

F = dm/dt * v

Rearranging the formula and substituting the given values:

dm/dt = F / v = 98 N / 390 m/s = 0.2513 kg/s

Therefore, the mass per second that must leave the rocket to develop the thrust of 98 N is 0.2513 kg/s.

To learn more about the moment of inertia click here

https://brainly.com/question/31045808

#SPJ11

The frequency of a sound wave is 457 Hz and the speed is
342.5 m/s. What is the sound's wavelength?

Answers

Answer:

/\ = 0.75m

Explanation:

v = f × /\

342.5= 457/\

/\ = 0.75m

In the circuit shown, serves as an electronic switch. If Vin is very small, determine W/ such that the circuit attenuates the signal by only 5%.Assume Va = 2.1Vand R = 1750. Assume an NMOS transistor with k'=800 A/V2 and VTh=0.5V

Answers

To attenuate the signal by only 5%, the width-to-length ratio (W/L) of the NMOS transistor should be approximately 13.125.

To determine the required W/L ratio, we need to calculate the voltage gain of the circuit and find the value of W/L that corresponds to a 5% attenuation.

First, let's analyze the circuit. We have an NMOS transistor connected in a common-source configuration, where Vin is the input voltage, Va is the gate voltage, and Vout is the output voltage.

               _______________

              |               |

  Vin -------|               |

              |               |

              |               |

              |     NMOS      |------ Vout

              |               |

              |               |

              |               |

              |_______________|

               

The voltage gain (Av) of the NMOS transistor in the common-source configuration can be approximated as:

Av = -k' * (W/L) * (Vin - VTh) * (1 + λ * Vout)

where k' is the transconductance parameter, W/L is the width-to-length ratio, Vin is the input voltage, VTh is the threshold voltage, λ is the channel-length modulation parameter, and Vout is the output voltage.

Since the question states that Vin is very small, we can assume that the voltage drop across R is negligible. Therefore, Va will be approximately equal to Vin.

Va = Vin

We want the circuit to attenuate the signal by only 5%. This means the output voltage (Vout) should be 95% of the input voltage (Vin).

Vout = 0.95 * Vin

Substituting Va = Vin and Vout = 0.95 * Vin into the voltage gain equation, we get:

0.95 * Vin = -k' * (W/L) * (Vin - VTh) * (1 + λ * 0.95 * Vin)

Rearranging the equation, we have:

-k' * (W/L) * (Vin - VTh) * (1 + λ * 0.95 * Vin) = 0.95 * Vin

Simplifying, we get:

-k' * (W/L) * (1 + λ * 0.95 * Vin) = 0.95

Since Vin is very small, we can assume that λ * 0.95 * Vin is negligible compared to 1. Therefore, we can approximate the equation as:

-k' * (W/L) = 0.95

Now, let's substitute the given values:

k' = 800 A/V^2

VTh = 0.5 V

-800 * (W/L) = 0.95

Solving for W/L, we find:

W/L = 0.95 / -800 ≈ -0.0011875

The W/L ratio cannot be negative, so we take the absolute value:

W/L ≈ 0.0011875

Finally, we can calculate the width (W) for a given length (L). Assuming L = 1 (arbitrary units), we have:

W = (W/L) * L ≈ 0.0011875 * 1 ≈ 0.0011875

To attenuate the signal by only 5%, the width-to-length ratio (W/L) of the NMOS transistor should be approximately 0.0011875 (or 13.125 if the length is assumed to be 1 unit).

To learn more about transistors, visit    

https://brainly.com/question/1426190

#SPJ11

If the velocity of an object is v=-5t+30, at what time does it change direction? Ot=6 O t=5 Ot=3 Ot=2 Ot=0

Answers

Option (A) t = 6 , is the correct option .

The object changes direction at t = 6. The given velocity equation is v = -5t + 30, where v represents velocity and t represents time.

To determine when the object changes direction, we need to find the time at which the velocity becomes zero. The given velocity equation is v = -5t + 30, where v represents velocity and t represents time.

Setting the velocity equation equal to zero, we have:

-5t + 30 = 0

To solve for t, we can isolate t by subtracting 30 from both sides of the equation:

-5t = -30

Next, divide both sides of the equation by -5 to solve for t:

t = -30 / -5

t = 6

The object changes direction at t = 6. At this time, the velocity becomes zero.

To know more about velocity ,visit:

https://brainly.com/question/80295

#SPJ11

Your name is Galileo Galilei and you toss a weight upward at 20 feet per second from the top of the Leaning Tower of Pisa (height 186 ft).
(a) Neglecting air resistance, find the weight's velocity as a function of time
(b) Find the height (in feet) of the weight above the ground as a function of time.
(c) Where and when will it reach its zenith?

Answers

The height (in feet) of the weight above the ground as a function of time will be given by the equation h = -16t² + 20t + 186. The weight will reach its zenith at t = 0.625 seconds at a height of 197.125 feet above the ground.

The given problem is a classic example of projectile motion where an object is thrown from a height and lands on the ground. The height (in feet) of the weight above the ground as a function of time will be given by the equation h = -16t² + 20t + 186, where h represents the height of the weight above the ground and t represents the time in seconds.The zenith is the highest point of the weight, i.e., the point where the weight stops moving upward and starts moving downward. To find the zenith, we need to find the time when the vertical component of the weight's velocity becomes zero, i.e., when it stops moving upwards. This can be found by differentiating the equation for height with respect to time and setting it equal to zero, which gives us the time when the vertical velocity is zero. This time is t = 0.625 seconds.

Know more about projectile motion, here:

https://brainly.com/question/12860905

#SPJ11

an airplane has a maximum velocity of 160km/h in still air. calculate its maximum velocity when it travels in air with a crosswind of 30km/h

Answers

Answer:

Velocity can be directly added or subtracted.

For example, if a boat has a velocity V in still water.

And now you put the boat in a river with a current that has a velocity V'

The total velocity of the boat in that river is just the addition of these two velocities.

Velocity in the river  = V + V'

Where the only tricky part is that the velocity is a vector, so you need to take in account the directions of each vector.

In this case, we have a plane with a maximum velocity of 160km, let's assume a direction for this velocity, let's say that is in the positive x-direction.

Then we can write the velocity in the vector form:

velocity  = (vel in x-axis, vel in y-axis)

The velocity of the plane can be written as:

v = (160km/h, 0)

Now we add a crosswind of 30km/h

crosswind means that it is perpendicular, then it acts on the y-axis.

Then the total velocity of the plane will be:

velocity = (160km/h, 0) + (0, 30km/h)

velocity = (160km/h, 30km/h)

Now you can compute the total velocity of the airplane as the module of that vector.

Remember that for a vector (x, y) the module is:

mod = √(x^2 + y^2)

Then the module of the velocity is:

v = √( (160km/h)^2 + (30km/h)^2) = 162.8 km/h

Light enters glass from air. The angle of refraction will be A) greater than the angle of incidence. B) equal to the angle of incidence.

Answers

(B) The angle of refraction when light enters glass from air will be equal to the angle of incidence. This is in accordance with Snell's law, which relates the angles and refractive indices of the media involved.

Determine what is the Snell's law?

According to Snell's law, the relationship between the angle of incidence (θ₁), the angle of refraction (θ₂), and the refractive indices of the two media is given by:

n₁ sin(θ₁) = n₂ sin(θ₂),

where n₁ and n₂ are the refractive indices of the initial medium (air) and the second medium (glass), respectively.

When light travels from air to glass, the refractive index of air (n₁) is smaller than the refractive index of glass (n₂). As a result, the sine of the angle of refraction (θ₂) will be smaller than the sine of the angle of incidence (θ₁), given that the angles are measured with respect to the normal.

Since sin(θ₂) < sin(θ₁), the only way for Snell's law to hold true is if θ₂ is smaller than θ₁. Therefore, the angle of refraction will be equal to the angle of incidence, option B.

To know more about Snell's law, refer here:

https://brainly.com/question/31432930#

#SPJ4

Two protons are aimed directly toward each other by a cyclotron accelerator with speeds of 2.20×105 m/s, measured relative to the earth.
Find the maximum electrical force that these protons will exert on each other.

Answers

The maximum electrical force that these protons will exert on each other is 2.48 x 10^-13 N.

The electrical force between two charged particles can be calculated using Coulomb's Law:

F = (k * q1 * q2) / r^2

Where:

F is the electrical force,

k is the electrostatic constant (k ≈ 8.99 x 10^9 N m^2/C^2),

q1 and q2 are the charges of the particles (in this case, both are protons, so each charge is q = 1.6 x 10^-19 C),

and r is the distance between the particles (assuming they are in contact, r ≈ 2 x 10^-15 m).

To find the maximum electrical force, we need to calculate the force when the protons are closest to each other. This occurs when they are just about to collide, so their separation distance is equal to the sum of their radii (r ≈ 2 x 10^-15 m).

Plugging the values into the formula, we have:

F = (8.99 x 10^9 N m^2/C^2) * (1.6 x 10^-19 C) * (1.6 x 10^-19 C) / (2 x 10^-15 m)^2

F ≈ 2.48 x 10^-13 N

The maximum electrical force that these protons will exert on each other is approximately 2.48 x 10^-13 N. This force arises due to the interaction between their positive charges and is inversely proportional to the square of the distance between them.

To know more about electrostatic visit :

https://brainly.com/question/17692887

#SPJ11

Which is denser?

The water in a swimming pool or a quarter (coin).

Answers

Answer:

The coin is denser than any of the liquids, and will sink through everything. The oil is the least dense liquid, so it will float on the water, and the syrup is the densest liquid, so it will sink below the water.

Explanation:

cute copy and paste? ☏ ♡ ☆⋆◦★◦⋆°*•°

. * . . ° . ● ° .

¸ . ★ ° :. . • ° . * :. ☆

° :. ° .☆ . ● .° °★

★ ★°★ . * . °☆ . ● . ★ ° . • ○ ● . ☆ ★ ° ☆ ¸. ¸ ★ . • ° . *

¸ . ★ ° :. :. . ¸ . ● ¸ ° ¸. * ● ¸ °☆

☆ °☆ . * ● ¸ . ★¸ .

. * . . ° . ● ° .

° :. ° . ☆ . . • . ● .° °★ Not sure how to copy and paste? Just right click your mouse and choose copy in options, to release repeat the process and just paste it. No mouse? Select the text with your computer pad and use ctrl c to release, ctrl v. On mobile? Press on your screen and select the text, use the option copy and paste wherever you would like!

You can use the information below to calculate it :)

Density of water: 1000 kg/m3.

Density of the coin: copper 8.96   g/cm^3

nickel   8.90   g/cm^3

1 kg = 1000 g

All you have to do now is convert it and thats it

When describing a thermodynamic system, which is a good description of "internal energy"?

Answers

In summary, the internal energy of a thermodynamic system is the sum of the potential and kinetic energy of all the particles in the system. It is a property of the system that depends only on the current state of the system and can be affected by a number of factors such as temperature, pressure, and composition of the system.

Internal energy is the energy that is associated with the microscopic components of the system, such as molecules and atoms. The internal energy of a thermodynamic system is the total potential energy and kinetic energy of all of the particles in the system. It includes the energy that is stored in the bonds between atoms and molecules and the kinetic energy of the individual particles. The internal energy of a thermodynamic system is a property of the system that depends only on the current state of the system. It can be increased or decreased by adding or removing heat or work from the system. "The internal energy of a system can be represented by the symbol U. "There are many factors that can affect the internal energy of a thermodynamic system. These include the temperature, pressure, and composition of the system, as well as any external forces that are acting on the system. The internal energy of a thermodynamic system is a key concept in the study of thermodynamics, as it helps to describe the behavior of systems as they undergo changes in temperature, pressure, and other variables.

to know more about Internal energy visit:

https://brainly.com/question/11278589

#SPJ11

Other Questions
PLS HELP ASAP What are the two factors that determine the boundaries of a biome?Question 1 options:A. temperatureB. latitudeC. precipitationD. altitude A nurse is performing a neurologic assessment on a 1-day-old neonate in the nursery. which findings indicate possible asphyxia in utero? Please help i will give brainliest and 5 * and 20 points for the best answerread the story and answer the two quetions1. What mood is conveyed by this excerpt?A. SuperiorB. DominantC. HopefulD. Heroic2. Read the following dictionary entryrestraint/rstrnt/ noun 1. ameasure or condition that keeps someone under control 2. depriving or restricting a person of freedom 3. a device which limits or prevents movement 4. self-controlWhich definition best matches the use of the word restraint?A. Definition 1B. Definition 2C. Definition 3D. Definition 4i don't want to see any linkif i do you will be reported Sara Awad File Home Insert Draw Design Layout References Mailings Review PROTECTED VIEW X Be careful files from the Internet can contain viruses. Unless you need to edit, it's safer to stay in Protected View. Enable Editing 7. Madden Company would like to estimate costs associated with its production of football helmets on a monthly basis. The accounting records indicate the following production costs were incurred last month for 4,000 helmets. Assembly workers' labor (hourly) $70,000 3,000 Factory rent Plant manager's salary 5,000 Supplies 20,000 Factory insurance 12,000 Materials required for production 20,000 Maintenance of production equipment (based on usage) 18,000 a. Identify which of the costs in the table above are fixed, and which are variable. (2.1) b. Use account analysis to estimate total fixed costs per month and the variable cost per unit. State your results in the cost equation form Y = f + vX by filling in the dollar amounts for f and v. (2.2) Estimate total production costs assuming 5,000 helmets will be produced and sold. (2.2) c. d. Prepare a contribution margin income statement assuming 5,000 helmets will be produced, and each helmet will be sold for $70. Fixed selling and administrative costs total $10,000. Variable , selling and administrative costs are $8 per unit. (5.1) Page 2 of 3 12 of 840 words Focus 10095 OLD Un 48 F Sunny View Help 0. X In a recent year, a research organization found that 517 of 766 surveyed male Internet users use social networking. By contrast 692 of 941 female Internet users use social networking. Let any difference refer to subtracting male values from female values. Complete parts a through d below. Assume that any necessary assumptions and conditions are satisfied. c) What is the standard error of the difference? (Round to four decimal places as needed.) d) Find a 95% confidence interval for the difference between these proportions. 00 (Round to three decimal places as needed.) In a recent year, a research organization found that 517 of 766 surveyed male Internet users use social networking. By contrast 692 of 941 female Internet users use social networking. Let any difference refer to subtracting male values from female values. Complete parts a through d below. Assume that any necessary assumptions and conditions are satisfied. c) What is the standard error of the difference? (Round to four decimal places as needed.) d) Find a 95% confidence interval for the difference between these proportions. 00 I need help pls and no files ok pls no files Delay in writing a claim letter makes the claim appear less important to the receiver.a. Trueb. False A quality control company was hired to study the length of meter sticks produced by a certain company. The team carefully measured the length of many meter sticks, and the distribution seems to be severely skewed to the right with a mean of 99.84 cm and a standard deviation of 0.2 cm. a) What is the probability of finding a meter stick with a length of more than 100.04 cm? ____ b) What is the probability of finding a group of 42 meter sticks with a mean length of less than 99.82 cm?_____ c) What is the probability of finding a group of 50 meter sticks with a mean length of more than 99.87 cm? _____d) What is the probability of finding a group of 28 meter sticks with a mean length of between 99.82 and 99.86 cm? ______e) For a random sample of 32 meter sticks, what mean length would be at the 92nd percentile? ______ The following macromolecules are not enzymatically digested in the stomach. a. proteins and carbohydrates b. carbohydrates and nucleic acids c. nucleic acids and fats d. fats and proteins Search using the internet on how you can expand or enhance one's knowledge of animal/fish raising. Two students in different classes took the same math test. Both students received ascore of 87. In student A's class the mean was 78 and the standard deviation of 5. Instudent B's class the mean was 76 with a standard deviation of 4. Which studentscored in the top 10% of their class? Determine the standard error of the estimated slope coefficient for the price of roses (point F) and whether that estimated slope coefficient is statistically significant at the 5 percent level. A. 9.42 and statistically significant since the t-statistic is greater than 2 in absolute value. B. 9.42 and statistically insignificant since the t-statistic is less than 2 in absolute value. C. 4.74 and statistically insignificant since the P-value is greater than 5 percent. D. 4.74 and statistically significant since the P-value is greater than 5 percent. Rubn, no quiero que tengas un mal da, pero es necesario que te (1)(decir) algo importante sobre tu audicin en el teatro. How many moles of hydrogenare in 3.06 x 10^-3 g of glycine C2H5NO2 6.01 x 0.2 = Can anyone do this plz At 45.0 C and a pressure of 9.9 kPa a sample of gas has a volume of 1.033 L. If the pressure is increased to 1245 kPa what will the new volume be? the book of esther was likely written to jews living ________ the land of israel. - What were the countries of Britain, France, Belgium, Russia, and Serbia called during WWI? what is a dynamic website? the person responsible for creating the original website content includes data that change based on user action information is stored in a dynamic catalog, or an area of a website that stores information about products in a database an interactive website kept constantly updated and relevant to the needs of its customers using a database unsaved change moving to the next question prevents changes to this answer. question 46 85) a particular triplet of bases in the template strand of dna is 5'-agt-3'. the corresponding codon for the mrna transcribed is a) 3'-uca-5'. b) 3'-uga-5'. c) 5'-tca-3'. d) 3'-acu-5'.